Antiderivative of $g(x)dg(x)$












0














I am reading a book by Shreve "Stochastic Calculus for Finance II" and after computing a stochastic integral $int_{0}^{T}W(t)dW(t)$ where $W(t)$ is a Brownian motion he compares it to the integral
$$int_{0}^{T}g(t)dg(t) = int_{0}^{T}g(t)g^prime (t)dt = 0.5g^2(T),$$
where $g(t)$ is a differentiable function with $g(0)=0$. I don't get the fact that $int g(t)g^prime (t)dt = 0.5g^2(t)$. For me the right hand side is equal to $int g(t)dt$, without the $g^prime (t)$ term. Thanks in advance.










share|cite|improve this question
























  • What’s the integral of x with respect to x?
    – Fede Poncio
    2 days ago










  • $frac{d}{dt}left(frac{1}{2}g^2(t)right)=g(t)g'(t)$.
    – AddSup
    2 days ago
















0














I am reading a book by Shreve "Stochastic Calculus for Finance II" and after computing a stochastic integral $int_{0}^{T}W(t)dW(t)$ where $W(t)$ is a Brownian motion he compares it to the integral
$$int_{0}^{T}g(t)dg(t) = int_{0}^{T}g(t)g^prime (t)dt = 0.5g^2(T),$$
where $g(t)$ is a differentiable function with $g(0)=0$. I don't get the fact that $int g(t)g^prime (t)dt = 0.5g^2(t)$. For me the right hand side is equal to $int g(t)dt$, without the $g^prime (t)$ term. Thanks in advance.










share|cite|improve this question
























  • What’s the integral of x with respect to x?
    – Fede Poncio
    2 days ago










  • $frac{d}{dt}left(frac{1}{2}g^2(t)right)=g(t)g'(t)$.
    – AddSup
    2 days ago














0












0








0







I am reading a book by Shreve "Stochastic Calculus for Finance II" and after computing a stochastic integral $int_{0}^{T}W(t)dW(t)$ where $W(t)$ is a Brownian motion he compares it to the integral
$$int_{0}^{T}g(t)dg(t) = int_{0}^{T}g(t)g^prime (t)dt = 0.5g^2(T),$$
where $g(t)$ is a differentiable function with $g(0)=0$. I don't get the fact that $int g(t)g^prime (t)dt = 0.5g^2(t)$. For me the right hand side is equal to $int g(t)dt$, without the $g^prime (t)$ term. Thanks in advance.










share|cite|improve this question















I am reading a book by Shreve "Stochastic Calculus for Finance II" and after computing a stochastic integral $int_{0}^{T}W(t)dW(t)$ where $W(t)$ is a Brownian motion he compares it to the integral
$$int_{0}^{T}g(t)dg(t) = int_{0}^{T}g(t)g^prime (t)dt = 0.5g^2(T),$$
where $g(t)$ is a differentiable function with $g(0)=0$. I don't get the fact that $int g(t)g^prime (t)dt = 0.5g^2(t)$. For me the right hand side is equal to $int g(t)dt$, without the $g^prime (t)$ term. Thanks in advance.







calculus integration self-learning stochastic-calculus






share|cite|improve this question















share|cite|improve this question













share|cite|improve this question




share|cite|improve this question








edited 2 days ago









J.G.

23.2k22137




23.2k22137










asked 2 days ago









tosik

1264




1264












  • What’s the integral of x with respect to x?
    – Fede Poncio
    2 days ago










  • $frac{d}{dt}left(frac{1}{2}g^2(t)right)=g(t)g'(t)$.
    – AddSup
    2 days ago


















  • What’s the integral of x with respect to x?
    – Fede Poncio
    2 days ago










  • $frac{d}{dt}left(frac{1}{2}g^2(t)right)=g(t)g'(t)$.
    – AddSup
    2 days ago
















What’s the integral of x with respect to x?
– Fede Poncio
2 days ago




What’s the integral of x with respect to x?
– Fede Poncio
2 days ago












$frac{d}{dt}left(frac{1}{2}g^2(t)right)=g(t)g'(t)$.
– AddSup
2 days ago




$frac{d}{dt}left(frac{1}{2}g^2(t)right)=g(t)g'(t)$.
– AddSup
2 days ago










2 Answers
2






active

oldest

votes


















1














You seem to be confusing $int g(t), dt$ with $int g(t), dg(t)$. It might help to make the substitution explicitly.



Let $y = g(t)$. Then $dg(t) = dy = g'(t) dt$, and so substituting $y=g(t)$ in the integral gives
$$int_{0}^{T} g(t), dg(t) = int_{t=0}^{t=T} y, dy = left[ dfrac{y^2}{2} right]_{t=0}^{t=T} = left[ dfrac{g(t)^2}{2} right]_0^T = dfrac{g(T)^2}{2}$$






share|cite|improve this answer





















  • You are right, I confused the integrals, thank you.
    – tosik
    2 days ago



















1














Just as $int g^prime dt=g+C$, $int f^prime(g)g^prime dt=f(g)+C$ by the chain rule. The case at hand is $f=g^2/2$.






share|cite|improve this answer





















    Your Answer





    StackExchange.ifUsing("editor", function () {
    return StackExchange.using("mathjaxEditing", function () {
    StackExchange.MarkdownEditor.creationCallbacks.add(function (editor, postfix) {
    StackExchange.mathjaxEditing.prepareWmdForMathJax(editor, postfix, [["$", "$"], ["\\(","\\)"]]);
    });
    });
    }, "mathjax-editing");

    StackExchange.ready(function() {
    var channelOptions = {
    tags: "".split(" "),
    id: "69"
    };
    initTagRenderer("".split(" "), "".split(" "), channelOptions);

    StackExchange.using("externalEditor", function() {
    // Have to fire editor after snippets, if snippets enabled
    if (StackExchange.settings.snippets.snippetsEnabled) {
    StackExchange.using("snippets", function() {
    createEditor();
    });
    }
    else {
    createEditor();
    }
    });

    function createEditor() {
    StackExchange.prepareEditor({
    heartbeatType: 'answer',
    autoActivateHeartbeat: false,
    convertImagesToLinks: true,
    noModals: true,
    showLowRepImageUploadWarning: true,
    reputationToPostImages: 10,
    bindNavPrevention: true,
    postfix: "",
    imageUploader: {
    brandingHtml: "Powered by u003ca class="icon-imgur-white" href="https://imgur.com/"u003eu003c/au003e",
    contentPolicyHtml: "User contributions licensed under u003ca href="https://creativecommons.org/licenses/by-sa/3.0/"u003ecc by-sa 3.0 with attribution requiredu003c/au003e u003ca href="https://stackoverflow.com/legal/content-policy"u003e(content policy)u003c/au003e",
    allowUrls: true
    },
    noCode: true, onDemand: true,
    discardSelector: ".discard-answer"
    ,immediatelyShowMarkdownHelp:true
    });


    }
    });














    draft saved

    draft discarded


















    StackExchange.ready(
    function () {
    StackExchange.openid.initPostLogin('.new-post-login', 'https%3a%2f%2fmath.stackexchange.com%2fquestions%2f3060938%2fantiderivative-of-gxdgx%23new-answer', 'question_page');
    }
    );

    Post as a guest















    Required, but never shown

























    2 Answers
    2






    active

    oldest

    votes








    2 Answers
    2






    active

    oldest

    votes









    active

    oldest

    votes






    active

    oldest

    votes









    1














    You seem to be confusing $int g(t), dt$ with $int g(t), dg(t)$. It might help to make the substitution explicitly.



    Let $y = g(t)$. Then $dg(t) = dy = g'(t) dt$, and so substituting $y=g(t)$ in the integral gives
    $$int_{0}^{T} g(t), dg(t) = int_{t=0}^{t=T} y, dy = left[ dfrac{y^2}{2} right]_{t=0}^{t=T} = left[ dfrac{g(t)^2}{2} right]_0^T = dfrac{g(T)^2}{2}$$






    share|cite|improve this answer





















    • You are right, I confused the integrals, thank you.
      – tosik
      2 days ago
















    1














    You seem to be confusing $int g(t), dt$ with $int g(t), dg(t)$. It might help to make the substitution explicitly.



    Let $y = g(t)$. Then $dg(t) = dy = g'(t) dt$, and so substituting $y=g(t)$ in the integral gives
    $$int_{0}^{T} g(t), dg(t) = int_{t=0}^{t=T} y, dy = left[ dfrac{y^2}{2} right]_{t=0}^{t=T} = left[ dfrac{g(t)^2}{2} right]_0^T = dfrac{g(T)^2}{2}$$






    share|cite|improve this answer





















    • You are right, I confused the integrals, thank you.
      – tosik
      2 days ago














    1












    1








    1






    You seem to be confusing $int g(t), dt$ with $int g(t), dg(t)$. It might help to make the substitution explicitly.



    Let $y = g(t)$. Then $dg(t) = dy = g'(t) dt$, and so substituting $y=g(t)$ in the integral gives
    $$int_{0}^{T} g(t), dg(t) = int_{t=0}^{t=T} y, dy = left[ dfrac{y^2}{2} right]_{t=0}^{t=T} = left[ dfrac{g(t)^2}{2} right]_0^T = dfrac{g(T)^2}{2}$$






    share|cite|improve this answer












    You seem to be confusing $int g(t), dt$ with $int g(t), dg(t)$. It might help to make the substitution explicitly.



    Let $y = g(t)$. Then $dg(t) = dy = g'(t) dt$, and so substituting $y=g(t)$ in the integral gives
    $$int_{0}^{T} g(t), dg(t) = int_{t=0}^{t=T} y, dy = left[ dfrac{y^2}{2} right]_{t=0}^{t=T} = left[ dfrac{g(t)^2}{2} right]_0^T = dfrac{g(T)^2}{2}$$







    share|cite|improve this answer












    share|cite|improve this answer



    share|cite|improve this answer










    answered 2 days ago









    Clive Newstead

    50.7k474133




    50.7k474133












    • You are right, I confused the integrals, thank you.
      – tosik
      2 days ago


















    • You are right, I confused the integrals, thank you.
      – tosik
      2 days ago
















    You are right, I confused the integrals, thank you.
    – tosik
    2 days ago




    You are right, I confused the integrals, thank you.
    – tosik
    2 days ago











    1














    Just as $int g^prime dt=g+C$, $int f^prime(g)g^prime dt=f(g)+C$ by the chain rule. The case at hand is $f=g^2/2$.






    share|cite|improve this answer


























      1














      Just as $int g^prime dt=g+C$, $int f^prime(g)g^prime dt=f(g)+C$ by the chain rule. The case at hand is $f=g^2/2$.






      share|cite|improve this answer
























        1












        1








        1






        Just as $int g^prime dt=g+C$, $int f^prime(g)g^prime dt=f(g)+C$ by the chain rule. The case at hand is $f=g^2/2$.






        share|cite|improve this answer












        Just as $int g^prime dt=g+C$, $int f^prime(g)g^prime dt=f(g)+C$ by the chain rule. The case at hand is $f=g^2/2$.







        share|cite|improve this answer












        share|cite|improve this answer



        share|cite|improve this answer










        answered 2 days ago









        J.G.

        23.2k22137




        23.2k22137






























            draft saved

            draft discarded




















































            Thanks for contributing an answer to Mathematics Stack Exchange!


            • Please be sure to answer the question. Provide details and share your research!

            But avoid



            • Asking for help, clarification, or responding to other answers.

            • Making statements based on opinion; back them up with references or personal experience.


            Use MathJax to format equations. MathJax reference.


            To learn more, see our tips on writing great answers.





            Some of your past answers have not been well-received, and you're in danger of being blocked from answering.


            Please pay close attention to the following guidance:


            • Please be sure to answer the question. Provide details and share your research!

            But avoid



            • Asking for help, clarification, or responding to other answers.

            • Making statements based on opinion; back them up with references or personal experience.


            To learn more, see our tips on writing great answers.




            draft saved


            draft discarded














            StackExchange.ready(
            function () {
            StackExchange.openid.initPostLogin('.new-post-login', 'https%3a%2f%2fmath.stackexchange.com%2fquestions%2f3060938%2fantiderivative-of-gxdgx%23new-answer', 'question_page');
            }
            );

            Post as a guest















            Required, but never shown





















































            Required, but never shown














            Required, but never shown












            Required, but never shown







            Required, but never shown

































            Required, but never shown














            Required, but never shown












            Required, but never shown







            Required, but never shown







            Popular posts from this blog

            An IMO inspired problem

            Management

            Has there ever been an instance of an active nuclear power plant within or near a war zone?